How Do You Express a Sequence Using Set Builder Notation?

Click For Summary
The discussion focuses on expressing the sequence {-8, -3, 2, 7, 12, 17, ...} using set builder notation. The proposed solution is {2 + 5y | y ∈ ℤ}, indicating that the sequence can be generated by varying y over all integers. Participants confirm that replacing the starting element (2) with any element from the set will still yield a valid expression. There is a reassurance that seeking confirmation of correctness is acceptable in this context. Overall, the conversation emphasizes understanding and validating mathematical notation.
reenmachine
Gold Member
Messages
514
Reaction score
9

Homework Statement



Build a notation for the set: ##\{ ... , -8 , -3 , 2 , 7 , 12 , 17 ,...\}##

Homework Equations



##2+5(0) = 2##
##2+5(1) = 7##
##2+5(-1) = -3##

etc...

The Attempt at a Solution



##\{ \ 2+5y \ | \ y \in \mathbb{Z} \ \}##

Take note that you could replace ##2## by any elements of the set and it would still work.

any thoughts appreciated!

thank you!
 
Physics news on Phys.org
reenmachine said:

Homework Statement



Build a notation for the set: ##\{ ... , -8 , -3 , 2 , 7 , 12 , 17 ,...\}##

Homework Equations



##2+5(0) = 2##
##2+5(1) = 7##
##2+5(-1) = -3##

etc...

The Attempt at a Solution



##\{ \ 2+5y \ | \ y \in \mathbb{Z} \ \}##

Take note that you could replace ##2## by any elements of the set and it would still work.

any thoughts appreciated!

thank you!

Looks fine... Do you have a question?
 
No , just wanted to be sure it was correct.I'm slowly getting better at those.

Is that forbidden in this section? If so , I was unaware of it.

thank you!
 
reenmachine said:
No , just wanted to be sure it was correct.I'm slowly getting better at those.

Is that forbidden in this section? If so , I was unaware of it.

thank you!
No, it's OK to get confirmation that you're on the right track.
 
Question: A clock's minute hand has length 4 and its hour hand has length 3. What is the distance between the tips at the moment when it is increasing most rapidly?(Putnam Exam Question) Answer: Making assumption that both the hands moves at constant angular velocities, the answer is ## \sqrt{7} .## But don't you think this assumption is somewhat doubtful and wrong?

Similar threads

  • · Replies 7 ·
Replies
7
Views
2K
  • · Replies 15 ·
Replies
15
Views
2K
  • · Replies 10 ·
Replies
10
Views
2K
  • · Replies 9 ·
Replies
9
Views
2K
  • · Replies 24 ·
Replies
24
Views
5K
Replies
13
Views
2K
Replies
16
Views
2K
  • · Replies 8 ·
Replies
8
Views
2K
  • · Replies 1 ·
Replies
1
Views
1K
  • · Replies 13 ·
Replies
13
Views
2K